Đến nội dung

minhducndc nội dung

Có 158 mục bởi minhducndc (Tìm giới hạn từ 06-05-2020)



Sắp theo                Sắp xếp  

#697374 Cho a,b,c >0 ;abc=1.Chứng minh: $\sum \frac{1}...

Đã gửi bởi minhducndc on 28-11-2017 - 17:43 trong Bất đẳng thức và cực trị

 Câu 7, Ta đặt  $(\frac{a}{a-1};\frac{b}{b-1};\frac{c}{c-1})= (x;y;z)$

Do $abc=1\Rightarrow \prod \frac{a}{a-1}= \prod \frac{1}{a-1}= (x-1)(y-1)(z-1)$

hay $xyz= (x-1)(y-1)(z-1)\Leftrightarrow x+y+z=xy+yz+zx+1$

Khi đó   $\sum \frac{a^{2}}{(a-1)^{2}}= x^{2}+y^{2}+z^{2}= (x+y+z)^{2}-2(xy+yz+zx)$

$= (x+y+z)^{2}-2(x+y+z-1)= (x+y+z)^{2}-2(x+y+z)+2\geq 1$

Q.E.D

            

 




#696805 Tìm GTNN của $P=\frac{x}{x^{2}+1}+...

Đã gửi bởi minhducndc on 19-11-2017 - 09:53 trong Bất đẳng thức và cực trị

Ap dụng bất đẳng thức Cauchy dạng Engel ta có

$\frac{x}{x^{2}+1}+\frac{y}{y^{2}+1}+\frac{z}{z^{2}+1}\geq \frac{(x+y+z)^{2}}{x^{3}+y^{3}+z^{3}+x+y+z}= \frac{1}{x^{3}+y^{3}+z^{3}+1}$

$\Rightarrow P\geq \frac{1}{x^{3}+y^{3}+z^{3}+1}+\frac{1}{9xyz}$

Có $\frac{1}{\sum x^{3}+1}+\frac{1}{30xyz}\geq \frac{4}{\sum x^{3}+30xyz+1}$

$\Rightarrow P\geq \frac{4}{\sum x^{3}+30xyz+1}+\frac{7}{90xyz}$

Mà  $x^{3}+y^{3}+z^{3}+24abc\leq (x+y+z)^{3}$$\Rightarrow a^{3}+b^{3}+c^{3}+30xyz\leq (x+y+z)^{3}+6xyz\leq 1+\frac{2}{9}= \frac{11}{9}$

Suy ra P>=39/10




#696780 $\frac{a^{3}}{(1+a)(1+b)}+\frac...

Đã gửi bởi minhducndc on 18-11-2017 - 18:12 trong Bất đẳng thức và cực trị

Vì BĐT thuần nhất nên chuẩn hóa: $a+b+c=3$

BĐT cần chứng minh trở thành: $\frac{a^2}{2a^2+(3-2a)^2}+\frac{b^2}{2b^2+(3-2b)^2}+\frac{c^2}{2c^2+(3-2c)^2}\leq 1$

Ta có đánh giá: $\frac{a^2}{2a^2+(3-2a)^2}\leq \frac{2a-1}{3}$$\Leftrightarrow 3(a-1)^2(4a-3)\geq 0$

Nếu $a,b,c\geq \frac{3}{4}$ thì đánh giá luôn đúng nên ta có: $\frac{a^2}{2a^2+(3-2a)^2}+\frac{b^2}{2b^2+(3-2b)^2}+\frac{c^2}{2c^2+(3-2c)^2}\leq \frac{2(a+b+c)-3}{3}=1$

Nếu $a,b,c<\frac{3}{4}$ thì: $\frac{a^2}{2a^2+(3-2a)^2}<\frac{1}{3}$

$\Rightarrow \frac{a^2}{2a^2+(3-2a)^2}+\frac{b^2}{2b^2+(3-2b)^2}+\frac{c^2}{2c^2+(3-2c)^2}<1$

Vậy BĐT được chứng minh:

Dấu '=' xảy ra khi: $a=b=c=1$

hình như bạn xét thiếu trg hợp nếu 2 số$\geq \frac{3}{4}$, 1số $< \frac{3}{4}$ thì sao




#696777 $2x^3+1=y^3$

Đã gửi bởi minhducndc on 18-11-2017 - 17:56 trong Số học

<=> $(y-1)(y^{2}-y+1)=2x^{3}$

Ta thấy $y^{2}-y+1$ lẻ $\forall y\in Z$, $2x^{3}$ chẵn $\forall x\in Z$ và $2x^{3}\vdots (y^{2}-y+1)$

$=> \begin{bmatrix}y^{2}-y+1=1 & \\y^{2}-y+1=-1 & \end{bmatrix}$

Đến đây tự giải tiếp.

chỗ này bạn sai rồi số chẵn có thể có nhiều ước lẻ khác mà




#696745 Tìm Min A= $x^2+14y^2+10z^2-4\sqrt{2y}$

Đã gửi bởi minhducndc on 17-11-2017 - 22:03 trong Bất đẳng thức và cực trị

bạn xem ở đây http://phuhienltt.vi...ntry_id/4372469




#696602 $\sum a$

Đã gửi bởi minhducndc on 14-11-2017 - 19:33 trong Bất đẳng thức và cực trị

Ta có $x^{4}+x+x\geq 3\sqrt[3]{x^{6}}= 3x^{2}$

Tương tự ta có $(x^{4}+y^{4}+z^{4})+2(x+y+z)\geq 3(x^{2}+y^{2}+z^{2})= (x^{2}+y^{2}+z^{2})^{2}$

$\Rightarrow 2(x+y+z)\geq 2(\sum x^{2}y^{2})$ (dpcm)




#696299 TOPIC thảo luận, trao đổi toán thi học sinh giỏi khối 10,11 .

Đã gửi bởi minhducndc on 10-11-2017 - 16:57 trong Chuyên đề toán THPT

Bài toán số 7 ( sưu tầm )

Tìm a để phương trình $\sqrt{a+\sqrt{a+sinx}}=sinx$ có nghiệm

Đặt $sinx= b$ $(b\in \begin{bmatrix} -1;1 \end{bmatrix})$phương trình trở thành

$\sqrt{a+\sqrt{a+b}}= b$(*)

Điều kiện $\left\{\begin{matrix} a+b\geq 0\\ b\geq 0 \end{matrix}\right.$

Khi đó  $(*)\Leftrightarrow a+\sqrt{a+b}= b^{2}$

           $\Leftrightarrow a+b+\sqrt{a+b}+\frac{1}{4}= b^{2}+b+\frac{1}{4}$

            $\Leftrightarrow (\sqrt{a+b}+\frac{1}{2})^{2}= (b+\frac{1}{2})^{2}$

            $\Leftrightarrow \sqrt{a+b}=b$ (vì $\sqrt{a+b};b> 0$)

            $\Leftrightarrow b^{2}-b-a= 0$ (**)

Có phương trình (*) có nghiệm $\Leftrightarrow pt(**)$ có nghiệm $\in \begin{bmatrix} 0;1 \end{bmatrix}$

Đến đây có thể vẽ đồ thị 2 hàm $(P):y=b^{2}-b;(d):y=a$




#696021 Cho $a,b,c>0$ và $a+b+c$=3. CMR $\sum...

Đã gửi bởi minhducndc on 03-11-2017 - 20:59 trong Bất đẳng thức và cực trị

Đặt bđt trên là A

$A$ $\geq \sum \frac{a^2}{a^2+2abc}$

$A$ $\geq \frac{(a+b+c)^2}{a^2+b^2+c^2+6abc}$

$A$ $\geq \frac{3}{a^2+b^2+c^2+6abc}$

Ta có $a+b+c=3$ <=> $3\geq 3\sqrt[3]{abc}$

<=> $abc\leq 1$ <=> $6abc\leq 6$

<=> $A$ $\geq \frac{9}{a^2+b^2+c^2+6}$

$a^2+b^2+c^2\geq \frac{(a+b+c)^2}{3}=3$

<=> $A$ $\geq \frac{9}{6+3}=1$

Dấu bằng xảy ra khi $a=b=c=1$

Trình bày vội, không được rõ ràng lắm.

Đoạn cuối bạn nhầm dấu rồi

Ta cần chứng minh $(a+b+c)^{2}\geq a^{2}+b^{2}+c^{2}+6abc$

$\Leftrightarrow ab+bc+ca\geq 3abc$

Ta có $abc\leq 1\Rightarrow \sqrt[3]{a^{2}b^{2}c^{2}}\geq abc$

mặt khác $ab+bc+ca\geq 3\sqrt[3]{a^{2}b^{2}c^{2}}\geq 3abc$




#695893 $\frac{x^2}{y}+\frac{y^2}{2...

Đã gửi bởi minhducndc on 31-10-2017 - 17:48 trong Bất đẳng thức - Cực trị

BĐT sai x=4;y=4,5;z=3

với điều kiện trên liệu có tìm được Min không nhỉ




#695789 Cho $1\leq a,b,c\leq 3$ và $a+b+c=6$

Đã gửi bởi minhducndc on 29-10-2017 - 20:23 trong Bất đẳng thức và cực trị

Bài toán đưa về : Cho $1\leq a;b;c\leq 3;a+b+c=6.$ Chứng minh cho $ab+bc+ca\geq 11$

Không mất tính tổng quát ta giả sử $1\leq a\leq b\leq c\leq 3$$\Rightarrow 1\leq a\leq 2$

Khi đó $ab+bc+ca= a(b+c)+bc= a(6-a)+bc$

Do $b\leq 3;c\leq 3$$\Rightarrow (b-3)(c-3)\geq 0\Leftrightarrow bc\geq 3(b+c)-9= 3(6-a)-9$

$\Rightarrow ab+bc+ca\geq a(6-a)+3(6-a)-9= -a^{2}+3a+9$$= 11-(a-1)(a-2)\geq 11$

(Vì$1\leq a\leq 2\Rightarrow (a-1)(a-2)\leq 0$)

$\Rightarrow dpcm$




#695656 CHỨNG MINH :$ \sum \left ( \frac{1+\sqrt{1...

Đã gửi bởi minhducndc on 27-10-2017 - 17:58 trong Bất đẳng thức và cực trị

Ta có    $VT=\frac{1+\sqrt{x^{2}+1}}{x}+\frac{1+\sqrt{y^{2}+1}}{y}+\frac{1+\sqrt{z^{2}+1}}{z}$

                  $= \frac{1}{xyz}(\sum yz+\sum yz\sqrt{x^{2}+1})$

Như vậy ta cần chứng minh

$\sum yz+\sum yz\sqrt{x^{2}+1}\leq x^{2}y^{2}z^{2}= (x+y+z)^{2}$

Có  $yz\sqrt{x^{2}+1}\leq \frac{1}{4}yz(x^{2}+5)$$\Rightarrow \sum yz+\sum yz\sqrt{x^{2}+1}\leq \sum yz+\frac{5}{4}\sum yz+\frac{1}{4}(x+y+z)xyz= \frac{9}{4}\sum yz+\frac{1}{4}(x+y+z)^{2}\leq \frac{3}{4}(x+y+z)^{2}+\frac{1}{4}(x+y+z)^{2}= (x+y+z)^{2}$

 




#695584 Tìm min P= $\frac{3a}{b+c}+\frac{4b...

Đã gửi bởi minhducndc on 26-10-2017 - 19:20 trong Bất đẳng thức và cực trị

Ta có 

$P=\frac{3a}{b+c}+\frac{4b}{c+a}+\frac{5c}{a+b}= \frac{3(a+b+c)}{b+c}+\frac{4(a+b+c)}{a+c}+\frac{5(a+b+c)}{a+b}-12$

   $= (a+b+c)(\frac{3}{b+c}+\frac{4}{a+c}+\frac{5}{a+c})-12$

Có $\frac{3}{b+c}+\frac{4}{a+c}+\frac{5}{a+b}\geq \frac{(\sqrt{3}+2+\sqrt{5})^{2}}{2(a+b+c)}$

$\Rightarrow P\geq \frac{(2+\sqrt{3}+\sqrt{5})^{2}}{2}$-12

Dâu bằng xảy ra khi$\frac{\sqrt{3}}{b+c}= \frac{2}{a+c}= \frac{\sqrt{5}}{a+b}$




#695495 $\frac{x}{y}+\frac{y}{x...

Đã gửi bởi minhducndc on 25-10-2017 - 21:04 trong Bất đẳng thức và cực trị

Đặt x+y-S;xy=P ta có $S^{2}\geq 4P$

$6S^{2}+8P=5S(P+3)\Rightarrow P= \frac{6S^{2}-15S}{5S-8}$

Ta cần chứng minh 

$\frac{S^{2}-2P}{P\geq }\frac{10}{3}\Leftrightarrow \frac{S^{2}}{P}\geq \frac{16}{3}$

thế P vào ta có $\frac{S^{2}(5S-8)}{6S^{2}-15S}\geq \frac{10}{3}\Leftrightarrow 5(S-4)^{2}\geq 0 (Do S\geq \frac{16}{5})$




#695470 $\frac{a^3}{b^2+3}+\frac{b^3}...

Đã gửi bởi minhducndc on 25-10-2017 - 19:37 trong Bất đẳng thức - Cực trị

$\sum \frac{a^{3}}{b^{2}+3}= \sum \frac{a^{3}}{b^{2}+ab+bc+ca}= \sum \frac{a^{3}}{(b+c)(b+a)}.$

Áp dụng AM-GM cho 3 số $: \frac{a^{3}}{(b+a)(b+c)}+\frac{b+a}{8}+\frac{b+c}{8} \geq \frac{3a}{4}.$

Tương tự ta có đpcm.




#695211 $\sum \frac{a}{b^2+bc}\leq \frac...

Đã gửi bởi minhducndc on 22-10-2017 - 11:00 trong Bất đẳng thức và cực trị

$\sum \frac{a}{b^{2}}\leq 1$ sai khi a=3;b=4;c=7




#695172 Chứng minh rằng $\sum \frac{(b+c)^{2}}...

Đã gửi bởi minhducndc on 21-10-2017 - 18:02 trong Bất đẳng thức và cực trị

Không mất tính tổng quát, giả sử a+b+c=1.

Bất đẳng thức cần chứng minh trở thành:

$\frac{(1-a)^{2}}{a(1+a)} + \frac{(1-b)^{2}}{b(1+b)} + \frac{(1-c)^{2}}{c(1+c)} \geq \frac{a}{1-a} + \frac{b}{1-b} + \frac{c}{1-c}$

Ta có:

$\frac{(1-a)^{2}}{a(1+a)} + \frac{9}{2}a(1-a) + \frac{3}{4}(1+a)\geq \frac{9}{2}(1-a)$

<=> $\frac{(1-a)^{2}}{a(1+a)}\geq \frac{9}{2}a^{2} - \frac{39}{4}a + \frac{15}{4}$

Tương tự suy ra:

$\frac{(1-a)^{2}}{a(1+a)} + \frac{(1-b)^{2}}{b(1+b)} + \frac{(1-c)^{2}}{c(1+c)} \geq \frac{9}{2}(a^{2} + b^{2} + c^{2}) - \frac{39}{4} (a +b +c) + \frac{45}{4} \geq \frac{3}{2}(a + b + c)^{2} - \frac{39}{4} (a +b +c) + \frac{45}{4} = 3$

Ta cần chứng minh vế phải $\leq 3$

$\frac{a}{1-a} + \frac{b}{1-b} + \frac{c}{1-c} \leq 3$

<=> $\frac{1}{a-1} + \frac{1}{b-1} + \frac{1}{c-1} \geq -\frac{9}{2}$

Áp dụng bất đẳng thức Cauchy:

$\frac{1}{a-1} + \frac{1}{b-1} + \frac{1}{c-1} \geq \frac{9}{a + b + c - 3} = -\frac{9}{2}$ (đpcm)

Dấu "=" xảy ra khi a = b = c.

Đoạn đánh giá cuối sai rồi bạn , $\sum \frac{1}{1-a}$ là số âm nên ko đánh giá vậy đc đâu

Với lại $\sum \frac{a}{b+c}\geq \frac{(a+b+c)^{2}}{2(ab+bc+ca)}\geq \frac{3}{2}$




#695171 CMR $\sum \frac{a+1}{-a^{2}+4a+1-bc...

Đã gửi bởi minhducndc on 21-10-2017 - 17:37 trong Bất đẳng thức và cực trị

Có $\frac{a+1}{-a^{2}+4a+1-bc}= \frac{a^{2}+a}{-a^{3}+4a^{2}+a-1}\geq \frac{1}{3a}+\frac{1}{3}$ (Vì $\frac{1}{2}\leq a\leq 4$)

Tương tự ta có $VT\geq \sum \frac{1}{3a}+1\geq 3\sqrt[3]{\frac{1}{27abc}}+1= 2$




#695124 $\frac{a^{3}}{(b+c)^{2}}+...

Đã gửi bởi minhducndc on 20-10-2017 - 18:01 trong Bất đẳng thức và cực trị

Ap dụng bất đẳng thức Cauchy cho 3 số dương ta có

$\frac{a^{3}}{(b+c)^{2}}+\frac{b+c}{8}+\frac{b+c}{8}\geq 3\sqrt[3]{\frac{a^{3}}{64}}= \frac{3a}{4}$

hay $\frac{a^{3}}{(b+c)^{2}}\geq \frac{3}{4a}-\frac{b+c}{4}$

Tương tự ta được đpcm

Hình như đề bài ko cho a,b,c dương




#695006 $\sum \frac{1}{a}\geq 4\sum a$

Đã gửi bởi minhducndc on 17-10-2017 - 21:12 trong Bất đẳng thức và cực trị

Ta có $ab+bc+ca+2abc=1\Rightarrow \frac{1}{a+1}+\frac{1}{b+1}+\frac{1}{c+1}= 2$ (Bạn quy đồng lên)

Do đó tồn tại x,y,z sao cho $\frac{1}{a+1}= \frac{x+y}{x+y+z};\frac{1}{b+1}= \frac{y+z}{x+y+z};\frac{1}{c+1}= \frac{x+z}{x+y+z}$

(Vì $\sum \frac{y+z}{x+y+z}= 2$ )

$\Rightarrow a=\frac{z}{x+y};b=\frac{x}{y+z};c= \frac{y}{x+z}$

Như vậy ta cần chứng minh $\frac{x+y}{z}+\frac{y+z}{x}+\frac{x+z}{y}\geq 4(\frac{z}{x+y}+\frac{x}{y+z}+\frac{y}{x+z})$

Dễ thấy $\frac{4z}{x+y}\leq \frac{z}{x}+\frac{z}{y}$

Làm tương tự với các số còn lại ta được đpcm




#694934 $(a+b+c)^{2}\geq 2(a+b+c)+ab+bc+ca$

Đã gửi bởi minhducndc on 16-10-2017 - 21:00 trong Bất đẳng thức và cực trị

Còn cách khác không anh




#694922 $(a+b+c)^{2}\geq 2(a+b+c)+ab+bc+ca$

Đã gửi bởi minhducndc on 16-10-2017 - 17:14 trong Bất đẳng thức và cực trị

Cho a,b,c>0 thỏa mãn $a^{2}+b^{2}+c^{2}+abc=4$.

Chứng minh rằng $(a+b+c)^{2}\geq 2(a+b+c)+ab+bc+ca$




#694921 $A=x^2y^3+y^2z^3+z^2x^3+(x-1)^2+(y-1)^2+(z-1)^2$

Đã gửi bởi minhducndc on 16-10-2017 - 17:08 trong Bất đẳng thức và cực trị

$A=x^2y^3+y^2z^3+z^2x^3+(x-1)^2+(y-1)^2+(z-1)^2=(x^2y^3+x+1)+(y^2z^3+y+1)+(z^2x^3+z+1)+x^2+y^2+z^2+x+y+z\geq 4(xy+yz+xz)+\sqrt{3(xy+yz+xz)}=15$

Vậy Min $A=15$.....

$(x-1)^{2}$ chứ có phải là $(x+1)^{2}$ đâu chị




#694889 Tìm GTLN

Đã gửi bởi minhducndc on 15-10-2017 - 20:35 trong Bất đẳng thức và cực trị

Có $B=\frac{3x^{2}-2x+3}{x^{2}+1}\Rightarrow 3x^{2}-2x+3-Bx^{2}-B=0$

hay $(3-B)x^{2}-2x+3-B=0$ (*)

Thấy pt (*) có nghiệm x $\Leftrightarrow \Delta \geq 0$$\Leftrightarrow (-2)^{2}-4(3-B)^{2}\geq 0$




#694879 $a,b,c>0$,$a+b+c=3$. CM: $\sum_{a,b,c...

Đã gửi bởi minhducndc on 15-10-2017 - 17:11 trong Bất đẳng thức và cực trị

 

Dấu '=' xảy ra khi: $(a;b;c)=(1;1;1)$ hoặc $(a;b;c)=(0;1;2)$

Cặp nghiệm $(a;b;c)= (0;1;2)$ không xảy ra dấu bằng mà bạn




#694732 $(a+b+c)(a+b+c-abc)\geq 2(a^{2}b+b^{2}c+c^{2}a)$

Đã gửi bởi minhducndc on 14-10-2017 - 10:02 trong Phương trình - Hệ phương trình - Bất phương trình

Cho các số thực không âm a,b,c thỏa mãn $$ a^{2}+b^{2}+c^{2} \leq 3$$ . Chứng minh rằng $(a+b+c)(a+b+c-abc)\geq 2(a^{2}b+b^{2}c+c^{2}a)$

Ta có bất đẳng thức quen thuộc $(a+b+c)(a^{2}+b^{2}+c^{2})\geq 3(a^{2}b+b^{2}c+c^{2}a)$

$\Rightarrow 3(a+b+c)\geq (a^{2}+b^{2}+c^{2})(a+b+c)\geq 3(a^{2}b+b^{2}c+c^{2}a)$

$\Rightarrow 2(a+b+c)\geq 2(a^{2}b+b^{2}c+c^{2}a)$

Như vậy ta cần chứng minh $a+b+c-abc\geq 2$ (*)

Theo đề bài ta có $a^{2}+b^{2}+c^{2}\leq 3\Rightarrow a+b+c\leq 3;abc\leq 1$

Có $a+b+c-abc\geq a+b+c-\frac{(a+b+c)^{3}}{27}$

Từ (*) ta cần chứng minh $a+b+c-\frac{(a+b+c)^{3}}{27}\geq 2\Leftrightarrow (a+b+c-3)^{2}(a+b+c+6)\geq 0$ (luôn đúng)